- Thu Jan 17, 2019 10:24 pm
#61893
Hi,
On page 305, Question 2, answerchoice (C),
Your explanation is "Some individuals taking antidepressants that cause weight gain will gain weight"
That is true, but I feel like this is not what the answerchoice indicates, which is basically "Some patients taking antidepressant drugs gain weight", and there seems to be no guarantee against the claim that patients who take antidepressant drugs will all choose antidepressant drugs that are not going to cause weight gain (a claim that is fair because the stimulus states "most" antidepressant drugs cause weight gain). So assuming that there are thirty patients who take antidepressant drugs, and 100 kinds of antideressant drugs, 51 of which cause weight gain, what if all thirty patients taking antidepressant drugs choose among 49? (Admittedly this scenario is unrealistic, but I think logic in LSAT must be bulletproof against all such scenarios)
In this scenario, I don't think you can equate "some patients taking antidepressant drugs" (in answer choice (C)) with "some individuals taking antidepressants that cause weight gain" (in your explanation).
So, my question is, is the stimulus assuming that Patients who take antidepressant drugs
antidepressant drugs?
Which is to say that all patients who take antidepressant drugs take antidepressant drugs, as well as all antidepressant drugs are taken by patients who take antidepressant drugs? I feel like only under such circumstance would this answer choice be correct, namely Patients who take antidepressant drugs
antidepressant drugs
gain weight which indicates Patients who take antidepressant drugs
gain weight,
without this assumption, Patients who take antidepressant drugs
antidepressant drugs
gain weight, which does not necessarily mean this answer choice is correct.
On page 305, Question 2, answerchoice (C),
Your explanation is "Some individuals taking antidepressants that cause weight gain will gain weight"
That is true, but I feel like this is not what the answerchoice indicates, which is basically "Some patients taking antidepressant drugs gain weight", and there seems to be no guarantee against the claim that patients who take antidepressant drugs will all choose antidepressant drugs that are not going to cause weight gain (a claim that is fair because the stimulus states "most" antidepressant drugs cause weight gain). So assuming that there are thirty patients who take antidepressant drugs, and 100 kinds of antideressant drugs, 51 of which cause weight gain, what if all thirty patients taking antidepressant drugs choose among 49? (Admittedly this scenario is unrealistic, but I think logic in LSAT must be bulletproof against all such scenarios)
In this scenario, I don't think you can equate "some patients taking antidepressant drugs" (in answer choice (C)) with "some individuals taking antidepressants that cause weight gain" (in your explanation).
So, my question is, is the stimulus assuming that Patients who take antidepressant drugs

Which is to say that all patients who take antidepressant drugs take antidepressant drugs, as well as all antidepressant drugs are taken by patients who take antidepressant drugs? I feel like only under such circumstance would this answer choice be correct, namely Patients who take antidepressant drugs



without this assumption, Patients who take antidepressant drugs

